Show that Euler-Mascheroni sequence is decreasing &monotonic

In summary, the conversation is discussing part a) of a homework problem involving the sequence tn=1+1/2+1/3+...+1/n - ln(n). The first step is to interpret tn - tn+1= [ln(n+1)-ln(n)] - 1/(n+1) as a difference of areas and show that tn - tn+1 > 0. The conversation then discusses different methods to prove this, including integrating both sides and comparing the areas of rectangles on a graph of f(x)=1/x.
  • #1
timnswede
101
0

Homework Statement


tn=1+1/2+1/3+...+1/n - ln(n)
a.) Interpret tn - tn+1= [ln(n+1)-ln(n)] - 1/(n+1) as a difference of areas to show that tn - tn+1 > 0.

Homework Equations

The Attempt at a Solution


I have not started working on part b) yet, because so far I am stuck on part a). I just simplified a bit and got ln(1+1/n)>1/(n+1). Not sure how to prove that the left side is bigger than the right.
 
Physics news on Phys.org
  • #2
Did you consider an integral?
There are other methods, too, depending on the equations you got for the logarithm.
 
  • #3
Is it as easy as just integrating both sides? I get xln(1+1/x)+ln(1+x)>ln(1+x). The left side is always greater since xln(1+1/x) is always greater than zero for n>1. Is that enough to prove it?
 
  • #4
Integrate both sides with respect to what?

[ln(n+1)-ln(n)] looks like an integration result.
 
  • #5
Woops those x's up there should be n's. But with dn. I can't see what integral would have resulted in [ln(n+1)-ln(n)] though. Integrating 1/n gets me ln(n), but integrating ln(n) does not get me ln(n+1).
 
  • #6
timnswede said:
Integrating 1/n gets me ln(n)
If you integrate from where to where?
 
  • #7
mfb said:
If you integrate from where to where?
From the beginning of the sequence, 1, to the end, n.
 
  • #8
timnswede said:
From the beginning of the sequence, 1, to the end, n.
What if you integrate it from n to n+1?
Also can you think of the area of rectangle formed with height 1/(n+1) and width (n+1)-n? Just compare them on the graph of f(x)=1/x
 

1. What is the Euler-Mascheroni sequence?

The Euler-Mascheroni sequence, also known as the gamma constant, is a mathematical constant that appears in many areas of mathematics, including number theory, analysis, and statistics. It is denoted by the symbol γ and has an approximate value of 0.5772156649.

2. How is the Euler-Mascheroni sequence defined?

The Euler-Mascheroni sequence is defined as the difference between the harmonic series and the natural logarithm of n, where n is a positive integer. This can be expressed as: γ = ∑ 1/k - ln(n), where k ranges from 1 to n.

3. Why is the Euler-Mascheroni sequence decreasing?

The Euler-Mascheroni sequence is decreasing because the difference between the harmonic series and the natural logarithm of n becomes smaller as n increases. This is due to the convergence of the harmonic series, which means that as n increases, the terms in the series become smaller, causing the difference to decrease.

4. How can I prove that the Euler-Mascheroni sequence is decreasing?

To prove that the Euler-Mascheroni sequence is decreasing, you can use mathematical induction. First, show that the sequence is decreasing for n=1. Then, assume that the sequence is decreasing for some value k, and prove that it is also decreasing for k+1. This will prove that the sequence is decreasing for all positive integers.

5. Is the Euler-Mascheroni sequence also monotonic?

Yes, the Euler-Mascheroni sequence is not only decreasing, but it is also monotonic. This means that the sequence is always decreasing and never increases. This can also be proven using mathematical induction.

Similar threads

  • Calculus and Beyond Homework Help
Replies
9
Views
713
  • Calculus and Beyond Homework Help
Replies
1
Views
232
  • Calculus and Beyond Homework Help
Replies
5
Views
1K
  • Calculus and Beyond Homework Help
Replies
7
Views
1K
  • Calculus and Beyond Homework Help
Replies
13
Views
952
  • Calculus and Beyond Homework Help
Replies
9
Views
1K
  • Calculus and Beyond Homework Help
Replies
4
Views
286
  • Calculus and Beyond Homework Help
Replies
1
Views
975
  • Calculus and Beyond Homework Help
Replies
6
Views
2K
  • Calculus and Beyond Homework Help
Replies
1
Views
275
Back
Top